Administración     

Olimpiadas de Matemáticas
Página de preparación y problemas

Selector
La base de datos contiene 1154 problemas y 775 soluciones.
OME Local
OME Nacional
OIM
OME Andalucía
Retos UJA
—20
Problema 1131
Sean $a$, $b$ y $n$ enteros positivos tales que $bn$ es divisor de $an-a+1$ y sea $\alpha=\frac{a}{b}$. Demostrar que, al dividir los números \[\lfloor\alpha\rfloor,\lfloor 2\alpha\rfloor,\ldots,\lfloor(n-1)\alpha\rfloor\] entre $n$ los restos resultantes son iguales a $1,2,\ldots,n-1$ en algún orden.

Nota. $\lfloor x\rfloor$ denota la parte entera de un número real $x$.

Sin pistas
Sin soluciones
info
Si crees que el enunciado contiene un error o imprecisión o bien crees que la información sobre la procedencia del problema es incorrecta, puedes notificarlo usando los siguientes botones:
Informar de error en enunciado Informar de procedencia del problema
Problema 1126
Consideramos 2024 números primos distintos $p_1,p_2,\ldots,p_{2024}$ tales que \[p_1+p_2+\ldots+p_{1012}=p_{1013}+p_{1014}+\ldots+p_{2024}.\] Sea $A=p_1p_2\cdots p_{1012}$ y $B=p_{1013}p_{1014}\cdots p_{2024}$. Demostrar que $|A-B|\geq 4$.
pistasolución 1info
Pista. Demuestra que $A-B$ es múltiplo de $4$ y que no puede ser igual a $0$.
Solución. Ninguno de los primos puede ser igual a $2$ ya que en tal caso una de las sumas sería par y la otra impar, luego podemos suponer que todos los primos son impares, es decir, son congruentes con $1$ o con $3$ módulo $4$. Pongamos que en los $1012$ primeros primos hay $m_1$ de ellos congruentes con $1$ y $m_3$ congruentes con $3$ módulo $4$, mientras que en los últimos $1012$ primos hay $n_1$ congruentes con $1$ y $n_3$ congruentes con $3$ módulo $4$. La igualdad del enunciado módulo $4$ se lee \[m_1+3m_3\equiv n_1+3n_3\ (\text{mod }4).\qquad (\star)\] Como $m_1+m_3=n_1+n_3=1012\equiv 0\ (\text{mod }4)$, tendremos que $n_1\equiv -n_3\ (\text{mod }4)$ y $m_1\equiv-m_3\ (\text{mod }4)$, lo que nos permite reescribir la congruencia $(\star)$ como $2m_3\equiv 2n_3\ (\text{mod }4)$. Esto nos dice que $m_3$ y $n_3$ tienen la misma paridad (¿por qué?). Por lo tanto, \[A\equiv 3^{m_3}\equiv 3^{n_3}\equiv B\ (\text{mod }4)\] ya que la potencia $3^a$ módulo $4$ es igual a $1$ si $a$ es par o a $3$ si $a$ es impar. Hemos probado así que $A-B$ es múltiplo de $4$ pero no puede ser igual a $0$ ya que $A\neq B$ (son todos primos distintos). Tenemos así que $|A-B|\geq 4$.
Si crees que el enunciado contiene un error o imprecisión o bien crees que la información sobre la procedencia del problema es incorrecta, puedes notificarlo usando los siguientes botones:
Informar de error en enunciado Informar de procedencia del problema
Problema 1125
Hallar todas las ternas de números primos $(p,q,r)$ tales que \[p^3+q^3+r^3-(pq+qr+rp)(p+q+r)\] es múltiplo de $pqr$.
pistasolución 1solución 2info
Pista. Demuestra que cualquiera de los tres primos divide a la suma o a la diferencia de los otros dos.
Solución. Vamos a comenzar analizando tres casos particulares:
  • Supongamos primero que los tres primos son iguales, luego \[\frac{p^3+q^3+r^3-(pq+qr+rp)(p+q+r)}{pqr}=-6.\] Por tanto, cualquier terna de primos iguales es solución el problema.
  • Supongamos ahora que dos de los primos son iguales, pongamos $p=q$ sin perder generalidad. Tras simplificar, nos queda \[\frac{p^3+q^3+r^3-(pq+qr+rp)(p+q+r)}{pqr}=\frac{r(r-2p)}{p^2}-5.\] Para que este número sea entero, se debe cumplir que $p$ divida a $r$ o bien a $r-2p$. En cualquier caso, $p$ debe dividir a $r$, luego $p=r$ por ser ambos primos. Esto nos dice que si dos de los primos son iguales, entonces lo son los tres.
  • Supongamos ahora que un primo es suma de los otros dos, pongamos $p=q+r$ sin perder generalidad. Por tanto, uno de ellos tiene que ser par y, salvo reordenarlos, tendremos que $p=q+2$ y $r=2$. Comprobamos fácilmente que, para tal elección de primos, se tiene que \[\frac{p^3+q^3+r^3-(pq+qr+rp)(p+q+r)}{pqr}=-5,\] luego tenemos todas las soluciones de la forma $(q+2,q,2)$ con $q$ y $q+2$ primos gemelos, así como sus posibles reordenaciones.

Resueltos estos casos, supondremos en lo que sigue que los tres primos son distintos y que ninguno es suma de los otros dos. Podemos agrupar todos los términos en los que sea posible sacar factor común $p$ y factorizar el resto de términos de la siguiente forma: \begin{align*} p^3+q^3+&r^3-(pq+qr+rp)(p+q+r)\\ &=p(p^2-pq-rp-q^2-3qr-r^2)+q^3+r^3-qr(q+r) \\ &=p(p^2-pq-rp-q^2-3qr-r^2)+(q+r)(q-r)^2. \end{align*} Si este número es divisible entre $p$, también ha de serlo $(q+r)(q-r)^2$, de donde obtenemos que $p$ divide a $q+r$ o a $q-r$. De la misma forma, se tiene que $q$ divide a $r+p$ o a $r-p$ y que $r$ divide a $p+q$ o a $p-q$. En otras palabras, cada primo divide a la suma o a la diferencia de los otros dos. Para encontrar las soluciones, vamos a usar la siguiente acotación básica:

$(\star)$ Si $x,y\in\mathbb{Z}$ verifican $x\mid y$, entonces $|x|\leq|y|$ o bien $y=0$.

Salvo reordenar los primos, tenemos cuatro posibilidades:

  • Si $p\mid (q+r)$, $q\mid (r+p)$ y $r\mid (p+q)$, entonces claramente $pqr\mid (p+q+r)$, luego podemos acotar $pqr\leq p+q+r\leq 3\max\{p,q,r\}$. Esto nos dice que dos de los primos tienen que tener producto menor o igual que $3$, lo cual es imposible.
  • Si $p\mid (q+r)$, $q\mid (r+p)$ y $r\mid (p-q)$, entonces podemos razonar de forma parecida al caso anterior. Teniendo en cuenta que $p-q+r\neq 0$ y $-p+q+r\neq 0$, podemos usar ($\star$) y la desigualdad triangular para obtener \begin{align*} pq|(p+q+r)&\Rightarrow pq\leq p+q+r\leq 3\max\{p,q,r\},\\ qr|(p-q+r)&\Rightarrow qr\leq |p-q+r|\leq |p|+|q|+|r|\leq 3\max\{p,q,r\},\\ pr|(-p+q+r)&\Rightarrow pr\leq |-p+q+r|\leq |p|+|q|+|r|\leq 3\max\{p,q,r\}. \end{align*} Esto nos dice que debe haber dos primos menores o iguales que $3$, es decir, uno de ellos tiene que ser $2$ y otro $3$. Sustituyendo sin perder generalidad $p=2$ y $q=3$ en la expresión original, se tiene \[\frac{p^3+q^3+r^3-(pq+qr+rp)(p+q+r)}{pqr}=\frac{r^3-5 r^2-31 r+5}{6 r}.\] Para que este último sea un número entero, tiene que ser $r=5$, luego obtenemos la solución $(p,q,r)=(2,3,5)$ y todas sus posibles reordenaciones.
  • Si $p\mid (q+r)$, $q\mid (r-p)$ y $r\mid (p-q)$, siguiendo la misma línea argumental anterior, como quiera que $p\neq q+r$, tenemos que \[pqr\mid (-p+q+r)\Rightarrow pqr\leq |-p+q+r|\leq p+q+r\leq 3\max\{p,q,r\},\] lo que de nuevo nos da que dos de los tres primos tiene producto menor o igual que $3$ y, por tanto, este caso no es posible.
  • Finalmente, supongamos que $p\mid (q-r)$, $q\mid (r-p)$ y $r\mid (p-q)$, luego \begin{align*} pq|(p+q-r)&\Rightarrow pq\leq |-p+q+r|\leq 3\max\{p,q,r\},\\ qr|(-p+q+r)&\Rightarrow qr\leq |-p+q+r|\leq 3\max\{p,q,r\},\\ pr|(p-q+r)&\Rightarrow pr\leq |p-q+r|\leq 3\max\{p,q,r\}. \end{align*} Esto nos dice que debe haber dos primos menores o iguales que $3$, es decir, uno de ellos tiene que ser $2$ y otro $3$, luego el tercero ha de ser $5$.

Resumiendo todos los casos anteriores, tenemos las soluciones de la forma $(p,p,p)$ y las de la forma $(2,p,p+2)$, salvo reordenaciones, siendo $p$ y $p+2$ primos gemelos.

Solución. Esta es una solución aportada por el profesor Samuel G. Moreno.

Sin pérdida de generalidad, supondremos que $p\leq q\leq r$. Si suponemos que la expresión del enunciado es igual a $\lambda pqr$ para cierto entero $\lambda$, esta relación se puede reescribir como \[(p+q+r)((q+r-p)^2-4qr)=(\lambda-3)pqr,\] de donde $qr$ divide a $(p+q+r)(q+r-p)^2=(q^2-p^2+r^2+2qr)(q-p+r)$, pero (al ser $q$ y $r$ primos) esto se traduce en que $r$ divide a $q+p$ o a $q-p$ y, además, que $q$ divide a $r-p$ o a $r+p$. Si fuese $q=p$, entonces la condición anterior se traduciría en que $qr$ dividiría a $(r^2+2qr)r$, lo que implicaría que $q$ dividiría a $r$, lo que sólo es posible si $r=q$. Esto nos da la primera solución \[(p,q,r)=(p,p,p)\qquad\text{para cualquier primo }p.\]

Además, si $p\lt q$, de modo que $1\leq q-p\lt r$, entonces no sería posible que $r$ divida a $q-p$. Surgen, por tanto, dos casos:

  • Caso $r\mid q+p$ y $q\mid r-p$ (siendo $p\lt q$). En este caso, existen enteros $m_1\gt 0$ y $m_2\geq 0$ tales que $p+q=m_1r$ y $r-p=m_2q$, de donde $(m_1-1)r=(1-m_2)q$.
    • Si $m_1=1$, entonces $m_2=1$. Como $p+q=m_1r=r$, al ser $r\gt p$, debe ser $r$ impar. No es posible que sea $p\geq 3$ ya que $q$ y $r$ serían ambos impares, de modo que $p=r-q$ sería un primo par y mayor o igual que $3$. Por tanto, debe ser $p=2$, lo que lleva a que $r=p+q=q+2$, que da la solución \[(p,q,r)=(2,q,q+2)\qquad\text{ siendo }q\text{ y }q+2\text{ primos gemelos.}\]
    • Si $m_1\gt 1$, sólo es posible que sea $m_2=0$ (ya que si $m_2\geq 1$, entonces $(1-m_2)q\leq 0$). Entonces, como $r-p=m_2q=0$, debemos tener $r=p$, que da $p=q$ (al ser $p\leq q\leq r$), contradiciendo una de las hipótesis de este caso.
  • Caso $r\mid q+p$ y $q\mid r+p$ (siendo $p\lt q$). En este caso existen enteros $m_1,m_2\geq 1$ con $p+q=m_1r$ y $r+p=m_2q$, de donde $(m_1+1)r=(m_2+1)q$.
    • Si $m_1=m_2=1$, entonces $2r=2q$, pero esto lleva a $p=m_1r-q=r-r=0$, lo que no es admisible.
    • Si $m_1\gt 1$, entonces $(m_1+1)r=(m_2+1)q$ nos da dos subcasos.
      • Si $r=q$, entonces $p=m_1r-q=(m_1-1)q$, que solo es posible si $m_1=2$, en cuyo caso volvemos a obtener la solución con los tres primos iguales (si fuera $m_1\geq 3$, entonces $p=(m_1-1)q$ no puede ser primo).
      • Si $r\neq q$, entonces $(m_1+1)=m_3q$ y $m_2+1=m_4r$ para ciertos enteros $m_3,m_4\geq 1$, en cuyo caso $p=m_1r-q=-r-q+m_3qr$ y $p=m_2q-p=-r-q+m_4qr$, de donde $p+q+r=m_0qr$, siendo $m_0=m_3=m_4$. De $p+q+r=m_0qr=(m_1+1)r$ y teniendo en cuenta que $m_1+1\geq 3$, tendríamos que $\frac{p+q+r}{3}=\frac{m_1+1}{3}r\geq r$, pero esto no es posible ya que la media de tres números distintos debe localizarse estrictamente entre el mayor y el menor de ellos.
    • Queda por analizar el caso $m_1=1$ y $m_2\gt 1$. En tal caso, se verifica $(m_1+1)r=2r=(m_2+1)q$, pero al ser $2\leq p\lt q$, entonces $q$ no puede dividir a $2$, pero tampoco a $r$. Esto significa que esta alternativa no es posible.
Si crees que el enunciado contiene un error o imprecisión o bien crees que la información sobre la procedencia del problema es incorrecta, puedes notificarlo usando los siguientes botones:
Informar de error en enunciado Informar de procedencia del problema
Problema 1122
Encontrar todos los números enteros positivos $n\lt 1000$ tales que las cuatro últimas cifras de $n^2$ pueden reordenarse para formar el número $2024$.
pistasolución 1info
Pista. Reduce primero el problema para ver que las únicas posibles cuatro últimas cifras de un cuadrado perfecto son $0224$ y $2204$.
Solución. Un cuadrado perfecto no puede terminar en $2$, lo que reduce considerablemente el número de posibles terminaciones. Además, como es par, las dos últimas cifras de $n^2$ deben formar un múltiplo de $4$, luego estas solo pueden ser $04$, $20$, $24$ o $40$, de las cuales podemos descartar $20$ y $40$ (ya que $n^2$ sería múltiplo de $10$ pero no de $100$). De aquí, obtenemos que las cuatro últimas cifras de $n^2$ solo pueden ser $2204$, $0224$ o $2024$. Podemos descartar $2024$ ya que en tal caso $n^2$ es múltiplo de $8$ pero no de $16$. También podemos descartar también $2204$, porque entonces $\frac{n^2}{4}$ sería un cuadrado que terminaría en $51$ luego la cifra de las unidades de $n$ sería $1$ o $9$. Sin embargo, $(10a+9)^2$ y $(10a+9)^2$ tienen la cifra de las decenas par para todo $a$ y esto nos dice que no hay cuadrados que terminen en $51$.

Para ver qué ocurre con $0224$, pongamos $n=100a+10b+c$, con $0\leq a,b,c\leq 9$ respectivamente. Podemos hacer la multiplicación siguiente: \[\begin{array}{ccccc} &&a&b&c\\ &\times&a&b&c\\\hline &&ac&bc&c^2\\ &ab&b^2&bc&\\ a^2&ab&ac&&\\\hline a^2&2ab&b^2\!+\!2ac&2bc&c^2 \end{array}\] y ahora ir cuadrando las cifras desde las unidades a las centenas. Esto es bastante rutinario, pero hay que tener cuidado de tener en cuenta las llevadas (no se han escrito en la multiplicación anterior ya que dependen de los valores concretos de $a,b,c$). En las unidades tenemos que $c^2\equiv 4\ (\text{mód } 10)$, con soluciones $c=2$ y $c=8$.

  • Si $c=2$, entonces en las decenas tenemos que $6b\equiv 2\ (\text{mód }10)$, que tiene soluciones $b=3$ y $b=8$. Para que $10b+c$ sea múltiplo de $4$, tiene que ser $b=3$, luego las centenas cuadran cuando $4a\equiv 2\ (\text{mód }10)$, que tiene soluciones $a=3$ y $a=8$. Tenemos que $332^2=110224$ sí cumple la condición pero $832^2=692224$ no.
  • Si $c=8$, entonces $6b\equiv 6\ (\text{mód }10)$, que tiene soluciones $b=1$ y $b=6$. Para que $10b+c$ sea múltiplo de $4$, tiene que ser $b=6$. Cuandrando las centenas, tenemos que $6a\equiv 6\ (\text{mód }10)$, que tiene soluciones $a=1$ y $a=6$. Sin embargo, las unidades de millar de $168^2=28224$ ni $668^2=446224$ no cuadran.
Hemos probado así que $n=332$ es la única solución al problema.
Si crees que el enunciado contiene un error o imprecisión o bien crees que la información sobre la procedencia del problema es incorrecta, puedes notificarlo usando los siguientes botones:
Informar de error en enunciado Informar de procedencia del problema
Problema 1112
Un número $n$ de siete cifras es bonito si se puede expresar como la suma de dos números de siete cifras $s$ y $t$ tales que todas las cifras de $s$ son impares y todas las cifras de $t$ son pares. Determinar cuáles de los siguientes números son bonitos: \[6204773, 6372538, 7343053, 8993267, 9652393.\]
pistasolución 1info
Pista. Si una cifra del número es par es porque hay una llevada en la suma de las cifras de orden inferior. Analiza cada número desde las unidades a las unidades de millón y te darás cuenta de que algunos son imposibles de expresar como $t+s$.
Solución. Hay que tener en cuenta que si una de las cifras del número es par es porque hay una llevada de la suma de las cifras de orden inmediatamente inferior. Vamos a analizar cada caso por separado:
  • 6204773 es bonito (por ejemplo, tomando $s=3557931$ y $t=2646842$).
  • 6372538 no es bonito puesto que la cifra de las unidades es par y no puede obtenerse como suma de un dígito par y otro impar (no hay llevadas).
  • 7343053 no es bonito ya que después del cero de las centenas hay necesariamente una llevada, luego la siguiente cifra no puede ser impar y en este caso es un tres.
  • 8993267 no es bonito ya que el 8 en las unidades de millón implica una llevada y por tanto las cifras de $t$ y $s$ en las centenas de millar deben sumar 19, lo que implica que ambas son 9 (más otra llevada de las decenas de millar). Esto impide que las cifras de $s$ sean todas pares.
  • 9652393 es bonito (tomando $t=5371751$ y $s=4280642$).
Si crees que el enunciado contiene un error o imprecisión o bien crees que la información sobre la procedencia del problema es incorrecta, puedes notificarlo usando los siguientes botones:
Informar de error en enunciado Informar de procedencia del problema
José Miguel Manzano © 2010-2024. Esta página ha sido creada mediante software libre